Chemistry, asked by bangtangirl1, 6 months ago

Please prove this...​

Attachments:

Answers

Answered by filter71
16

Answer:

refer to the attachment......

hope it helps u

Attachments:
Answered by Anonymous
28

Solution:

\\

First , we will convert every term of LHS into sin \theta and cos \theta.

 \\

LHS:

(\sec\theta -  \csc\theta)(1 +  \tan\theta +  \cot\theta) \:  \:  \:  \:  \:  \:  \:  \:  \:  \:  \:  \:  \:  \:  \:  \:  \:  \:  \:  \:  \:  \:  \:  \:  \:  \:  \:  \:  \:  \:  \:  \:  \:  \:  \:  \:  \:  \:  \:  \:  \:  \:  \:  \:  \:  \:  \:  \:  \:  \:  \:  \:  \:  \:  \:  \:  \:  \:  \:  \:   \:  \:  \:  \:  \:  \:  \:  \:  \: \:  \:  \:  \:  \: \:  \:  \:  \:  \:  \:  \:  \:  \:  \:  \:  \:  \:  \:  \:  \:  \:  \:  \:  \:  \:  \:  \:  \: \:  \:  \\  \\ \left( \sec\theta + \sec\theta\tan\theta + \sec\theta\cot\theta -  \csc\theta - \csc\theta\tan\theta - \csc\theta\cot\theta \right) \:  \:  \:  \:  \:  \:  \:  \:  \:  \:  \:  \:  \:  \:  \:  \:  \:  \:  \:  \:  \:  \:  \:  \:  \:  \:  \:  \:  \:  \:  \:  \:  \:  \\  \\ \left( \dfrac{1}{\cos\theta}  +  \dfrac{1}{\cos\theta} \times  \frac{\sin\theta}{\cos\theta} +  \dfrac{1}{\cos\theta}  \times  \dfrac{\cos\theta}{\sin\theta}  -  \dfrac{1}{ \sin\theta } -  \frac{1}{\sin\theta}   \times  \frac{\sin\theta}{\cos\theta}  -  \frac{1}{\sin\theta}  \times  \frac{\cos\theta}{\sin\theta}  \right) \\  \\

\left( \dfrac{1}{\cos\theta}  +  \dfrac{1}{\cos\theta} \times  \dfrac{\sin\theta}{\cos\theta} +  \dfrac{1}{\cancel{\cos\theta}}  \times  \dfrac{\cancel{\cos\theta}}{\sin\theta}  -  \dfrac{1}{ \sin\theta } -  \frac{1}{\cancel{\sin\theta}}   \times  \dfrac{\cancel{\sin\theta}}{\cos\theta}  -  \dfrac{1}{\sin\theta}  \times  \dfrac{\cos\theta}{\sin\theta}  \right) \\  \\ \left( \cancel{\dfrac{1}{\cos\theta} } - \cancel{ \dfrac{1}{\cos\theta}} +  \dfrac{\sin\theta}{\cos^{2} \theta} + \cancel{\dfrac{1}{\sin\theta} } - \cancel {\dfrac{1}{ \sin\theta }}  -  \dfrac{\cos\theta}{\sin^{2} \theta}  \right) \:  \:  \:  \:  \:  \:  \:  \:  \:  \:  \:  \:  \:  \:  \:  \:  \:  \:  \:  \:  \:  \:  \:  \:  \:  \:  \:  \:  \:  \:  \:  \:  \:  \:  \:  \:  \:  \:  \:  \:  \:  \:  \:  \:  \:  \:  \:  \:  \:  \:  \:  \:  \:  \:  \:  \:  \:  \:  \:  \:  \:  \:  \\  \\ \left( \dfrac{\sin\theta}{\cos\theta} \times  \dfrac{1}{\cos\theta}  -  \dfrac{\cos\theta}{\sin\theta}  \times  \dfrac{1}{\sin\theta}  \right) \:  \:  \:  \:  \:  \:  \:  \:  \:  \:  \:  \:  \:  \:  \:  \:  \:  \:  \:  \:  \:  \:  \:  \:  \:  \:  \:  \:  \:  \:  \:  \:  \:  \:  \:  \:  \:  \:  \:  \:  \:  \:  \:  \:  \:  \:  \:  \:  \:  \:  \:  \:  \:  \:  \:  \:  \:  \:  \:  \:  \:  \:  \:  \:  \:  \:  \:  \:  \:  \:  \:  \:  \:  \:  \:  \:  \:  \:  \:  \:  \:  \:  \:  \:  \:  \:  \:  \:  \:  \:  \:  \:  \:  \:  \:  \:  \:  \:  \: \\  \\

(\tan\theta\sec\theta - \cot\theta\csc\theta)  \\  \\

LHS = RHS

\\

Hence Proved.

Similar questions